Difference between revisions of "2014 USAJMO Problems/Problem 2"
(→Solution) |
(→Diagram) |
||
Line 6: | Line 6: | ||
(b) Line <math>OH</math> intersects segments <math>AB</math> and <math>AC</math> at <math>P</math> and <math>Q</math>, respectively. Denote by <math>s</math> and <math>t</math> the respective areas of triangle <math>APQ</math> and quadrilateral <math>BPQC</math>. Determine the range of possible values for <math>s/t</math>. | (b) Line <math>OH</math> intersects segments <math>AB</math> and <math>AC</math> at <math>P</math> and <math>Q</math>, respectively. Denote by <math>s</math> and <math>t</math> the respective areas of triangle <math>APQ</math> and quadrilateral <math>BPQC</math>. Determine the range of possible values for <math>s/t</math>. | ||
==Solution== | ==Solution== | ||
− | + | <asy> | |
+ | import olympiad; | ||
+ | unitsize(1inch); | ||
+ | pair A,B,C,O,H,P,Q,i1,i2,i3,i4; | ||
− | + | //define dots | |
+ | A=3*dir(50); | ||
+ | B=(0,0); | ||
+ | C=right*2.76481496; | ||
− | + | O=circumcenter(A,B,C); | |
+ | H=orthocenter(A,B,C); | ||
+ | |||
+ | i1=2*O-H; | ||
+ | i2=2*i1-O; | ||
+ | i3=2*H-O; | ||
+ | i4=2*i3-H; | ||
+ | //These points are for extending PQ. DO NOT DELETE! | ||
+ | |||
+ | P=intersectionpoint(i2--i4,A--B); | ||
+ | Q=intersectionpoint(i2--i4,A--C); | ||
+ | |||
+ | //draw | ||
+ | dot(P); | ||
+ | dot(Q); | ||
+ | draw(P--Q); | ||
+ | dot(A); | ||
+ | dot(B); | ||
+ | dot(C); | ||
+ | dot(O); | ||
+ | dot(H); | ||
+ | draw(A--B--C--cycle); | ||
+ | |||
+ | //label | ||
+ | label("$A$",A,N); | ||
+ | label("$B$",B,SW); | ||
+ | label("$C$",C,SE); | ||
+ | label("$P$",P,NW); | ||
+ | label("$Q$",Q,NE); | ||
+ | label("$O$",O,N); | ||
+ | label("$H$",H,N); | ||
+ | //change O and H label positions if interfering with other lines to be added | ||
+ | |||
+ | //further editing: ABCPQOH are the dots to be further used. i1,i2,i3,i4 are for drawing assistence and are not to be used | ||
+ | </asy> |
Revision as of 18:21, 30 April 2014
Problem
Let be a non-equilateral, acute triangle with $\angle A=60\textdegrees$ (Error compiling LaTeX. Unknown error_msg), and let and denote the circumcenter and orthocenter of , respectively.
(a) Prove that line intersects both segments and .
(b) Line intersects segments and at and , respectively. Denote by and the respective areas of triangle and quadrilateral . Determine the range of possible values for .
Solution